LSAT and Law School Admissions Forum

Get expert LSAT preparation and law school admissions advice from PowerScore Test Preparation.

User avatar
 Dave Killoran
PowerScore Staff
  • PowerScore Staff
  • Posts: 5853
  • Joined: Mar 25, 2011
|
#27058
Complete Question Explanation
(The complete setup for this game can be found here: lsat/viewtopic.php?t=3418)

The correct answer choice is (D)

As always, apply each rule to all of the contending answer choices in order to solve a List question:

Answer choice (A) is incorrect because neither J nor K is selected, a violation of the first rule.

Answer choice (B) is incorrect because Q is selected but K is not selected, a violation of the last rule (the lack of K also leads to the pairing of J and Q, a violation of the inference made during the setup).

Answer choice (C) is incorrect because N is selected but L is not selected, a violation of the third rule. This answer also pairs J and Q, a violation of the inference made during the setup.

Answer choice (E) is incorrect because both J and K are selected, a violation of the first rule. Note how answer choices (A) and (E) play on both sides of the first rule. Many students will eliminate one of the two answers, but then forget to check the “other” side.

Thus, answer choice (D) is proven correct by process of elimination.

Get the most out of your LSAT Prep Plus subscription.

Analyze and track your performance with our Testing and Analytics Package.